Exam Master questions

Pataasin ang iyong marka sa homework at exams ngayon gamit ang Quizwiz!

A 43-year-old woman who works as a secretary and spends most of her day typing on the computer has symptoms of carpal tunnel syndrome. Question What test is most useful in confirming the most likely diagnosis? 1 Electromyogram 2 Computerized tomogram (CT) of the wrist 3 Magnetic resonance image (MRI) of the wrist 4 Plain radiographs of the wrist 5 Erythrocyte sedimentation rate (ESR)

1 Electromyogram

A 37-year-old man fell from a ladder as he finished working on the roof of his house. The right side of his head hit the alley cement, and he lost consciousness for about 1 minute; he woke up with a headache, but he had no other complaints. A few hours later, the patient is brought to the emergency room by his neighbor because of an intense headache, confusion, and left hand hemiparesis. On examination, the patient has a bruise located over the right temporal region, mydriasis, and right deviation of the right eye, papilledema, and left extensor plantar response. An emergency CT scan of the head without contrast reveals a lens-shaped hyper-density under the right temporal bone with mass effect and edema. Question What is the most likely diagnosis? 1 Epidural hematoma 2 Subdural hematoma 3 Subarachnoid hemorrhage 4 Intracerebral parenchymal hemorrhage 5 Acute meningitis

1 Epidural hematoma Epidural hematoma most often results from a traumatic tear of the middle meningeal artery. Although a lucid interval ranging from minutes to hours followed by altered mental status and focal deficits is typical for epidural hematoma, this clinical picture is only encountered in up to 1/3 of patients. The collection of blood between the skull and dura mater causes an evident mass effect with ophthalmic nerve palsy and the contralateral hemiparesis. Surgical evacuation of the clot via burr holes is the treatment of choice.

A 32-year-old woman with a history of a tremor presents seeking help. She has experienced emotional stress from work over the past 6 months. Upon physical exam, a tremor is observed in her right hand when hands are outstretched. Tremor is also present in her head. Laboratory findings showed no abnormal findings, but her family history reveals that her father was also diagnosed with a tremor around the same age. Question What is the most likely explanation of the findings? 1 Essential tremor 2 Huntington's disease 3 Idiopathic torsion dystonia 4 Multiple sclerosis 5 Parkinsonism

1 Essential tremor The clinical picture is suggestive of essential tremor. A positive family history is commonly found due to responsible genes at 3q13, 2p22-p25, and 6p23. Essential tremors can be brought on by emotional stress and are most commonly a postural action tremor rather than a resting tremor.

A 22-year-old Asian man with no significant past medical history presents with a dull pain. It is insidious in onset and has been felt deep in the lower lumbar gluteal region for 5 months. It is accompanied by low-back morning stiffness that lasts up to a few hours. The stiffness improves with activity and returns with inactivity. For 2 months, the pain has ascended; it has become more persistent and bilateral. He notes that nocturnal exacerbation of pain forces him to rise and move around. He admits to bilateral hip and ankle pain, anorexia, malaise, and intermittent low-grade fever. Physical exam reveals a stooped forward-flexed position, limitation of anterior and lateral flexion, limited extension of the lumbar spine, positive Schober test, restricted ranges of motion of the hips, and reduced chest expansion. Lower back and hip x-ray shows bilateral SI joint erosion with narrowing of the joints. Question What additional clinical manifestation is most likely in this patient? 1 Eye pain and conjunctival injection 2 Hematuria and dysuria 3 Positive straight leg raise test 4 Subcutaneous nodule formations 5 Warm and swollen hip joints

1 Eye pain and conjunctival injection Eye pain and conjunctival injection are most likely to manifest in this patient who most likely has ankylosing spondylitis (AS). The most common extra-articular manifestation is acute anterior uveitis, which occurs in 40% of patients and can antedate the spondylitis. Attacks are typically unilateral, causing pain, photophobia, and increased lacrimation. These tend to recur, often in the opposite eye. Cataracts and secondary glaucoma are not uncommon sequelae. Up to 60% of patients have (usually asymptomatic) inflammation in the colon or ileum. Frank IBD occurs in 5-10% of patients with AS.

A 32-year-old man is admitted to the hospital following loss of consciousness. The patient had been ill with fever and headache for several days, then developed double vision, confusion, and loss of consciousness. He has been otherwise healthy with no past medical conditions. Imaging reveals edema of the frontal lobe. Question What virus most likely caused this patient's infection? 1 Herpes simplex virus 2 Adenovirus 3 Influenza A 4 West Nile virus 5 Epstein-Barr virus

1 Herpes simplex virus The patient has encephalitis, which is typically viral in nature. Herpes simplex virus is the most common viral cause of encephalitis.

A 48-year-old male patient positive for HIV starts to develop headaches. At first, he attributes the headaches to stress, but they persist and become worse over the next few weeks. He develops nausea and vomiting, and he thinks he has a fever. He starts to become confused, so he seeks medical attention. On physical examination, temperature is 100°F. Question What special maneuver of the neurological exam should be performed to support your diagnosis? 1 Kernig's sign 2 Babinski sign 3 Kussmaul's sign 4 Quincke's sign 5 Chvostek sign

1 Kernig's sign Kernig's sign is a sign of meningeal irritation. This patient has signs and symptoms of cryptococcal meningitis. Cryptococcus neoformans is an opportunistic yeast infection found in immunocompromised patients, which includes HIV-positive patients. To elicit Kernig's sign, the patient is placed in the supine position with knees bent. The examiner then extends (straightens) the leg. In the presence of meningeal irritation, leg straightening will elicit resistance and the patient will have pain in the lower back or thigh. Brudzinski sign is another special maneuver performed as part of the neurological examination to indicate meningeal irritation. In this maneuver, the clinician passively flexes the neck, which will result in pain and flexion of the hips and knees.

A 13-year-old girl presents for her school physical. On examination, you notice the posterior curvature of her thoracic spine to be very prominent and bulging backward. Question What type of deformity of the spine does she have? 1 Kyphosis 2 Scoliokyphosis 3 Scoliosis 4 Lordosis 5 Gibbus

1 Kyphosis

A 24-year-old man presents with head trauma received in a motor vehicle accident (MVA) 30 minutes ago. Paramedics relate that the patient experienced a loss of consciousness for 1 minute with a complete recovery. At the subsequent evaluation, the physical exam reveals a patient with a Glasgow Coma Scale (GCS) of 12, right hemiparesis, and a left fixed dilated pupil. Question What is the most probable finding in the brain CT scan? 1 Left epidural hematoma 2 Right epidural hematoma 3 Left subdural hematoma 4 Right subdural hematoma 5 Subarachnoid hemorrhage

1 Left epidural hematoma The patient presents with a history of head trauma, a brief initial period of unconsciousness, a lucid interval lasting minutes, subsequent deterioration in neurological status revealed in a Glasgow coma scale of 12, right hemiparesis, and left fixed dilated pupil. This is a classical presentation of left epidural hematoma, which is seen as a biconvex hyper-density (whiteness) on a CT scan.

An 82-year-old man with rheumatoid arthritis, HIV, and diabetes mellitus presents with fever, severe pain and swelling to his right knee. He denies any history of trauma, injuries or accidents, sexual activity, or prior episodes. On exam, right knee is erythematous with increased warmth. A joint aspiration was done which reveals opaque yellow synovial fluid with 55,000 WBCs/microL. Question What is the most likely additional manifestation in this patient? 1 Limited range of joint motion 2 Numbness and tingling 3 Positive McMurray's sign 4 Polyarticular joint pain 5 Erythema migrans rash

1 Limited range of joint motion This patient's presentation is most consistent with septic arthritis. The joint aspiration of septic arthritis typically reveals opaque-to-turbid synovial fluid with bacteria, leukocytosis, and high lactate levels. Clinically, patients present with severe pain, effusion, and resistance to joint motion. Patients with an infected joint generally present with triad of fever (40-60% of cases), pain (75% of cases), and impaired range of motion. Findings of overlying cellulitis, such as warmth and redness of the surrounding skin, may be present. These symptoms may evolve in a few days or a few weeks. Fever is usually low-grade (under 102°F), with rigors present in only 20% of cases. Unless an osteomyelitis in proximity is also present, plain radiographs and MRI scanning will reveal only a joint effusion. Arthrocentesis demonstrates an exudative effusion, with a leukocyte count over 50,000 WBCs/microL. Bacterial cultures are frequently positive, with Staphylococcus species the most frequent causative organisms.

An 18-year-old woman presents with a history of recurrent headaches that mainly occur above her right eye. She describes them as "throbbing" in quality. During these headaches, she gets bouts of nausea and vomiting. After a nap, she typically feels better. The headaches only last a few hours, and she experiences them roughly 2 times a month. Question What is the most likely diagnosis? 1 Migraine 2 Sinusitis 3 Cluster headache 4 Tension headache 5 Intracranial mass

1 Migraine

A 50-year-old man was playing baseball with his company team last weekend and is now experiencing severe pain in his left shoulder. He states that he has been the team pitcher for several years now. He has developed pain progressively in his left shoulder over the last few months. He denies any numbness or tingling in his arm, hand, or fingers of his left upper extremity. Question Which exam would you expect to yield pain? 1 Neer test 2 Tinel sign 3 Phalen test 4 Bragard stretch test 5 Apley grind test

1 Neer test Neer test is the only exam listed that evaluates for a rotator cuff injury. The Neer test is performed by having the patient internally rotate the shoulder while the practitioner flexes the arm forward. Pain will be elicited as the supraspinatus muscle presses against the acromion.

A 54-year-old man presents with acute onset of excruciating pain in his right toe. The patient states the pain began shortly after dinner, and it has progressively worsened since then to the point where he is now unable to bear weight on the affected side. In addition, he explains that aside from occasional backaches, he has never experienced pain like this before. He reports overall good health and aside from a multivitamin, he uses no medications or supplements. On exam, the patient's right foot is swollen, and the joint of the great toe is tense and inflamed. His temperature is 37°C, blood pressure is 155/85 mm Hg, and pulse is 103 beats per minute. Labs reveal an elevated serum uric acid level. Question What is examination of synovial fluid from the affected joint space most likely to reveal? 1 Negatively birefringent needle-shaped crystals 2 Positively birefringent needle-shaped crystals 3 Brownish clumps with elevated WBCs and negative bacteria 4 Increased polymorphonuclear leukocytes, WBCs, and positive bacteria 5 Decreased polymorphonuclear leukocytes, WBCs, and negative bacteria

1 Negatively birefringent needle-shaped crystals

A 4-year-old boy currently hospitalized with pneumonia develops vomiting and stiff neck. The new symptoms are followed by a seizure. On physical examination, he is febrile and has signs of meningeal irritation. A lumbar puncture is done to determine if he has meningitis. Test Results Normal Range CSF pressure. 238 mm H2O. 70-80 mm H2O. CSF leukocytes 8200 mm. 30-10 mm3 CSF glucose 26 mg/dL. 45-80 mg/dL CSF protein 253 mg/dL 15-45 mg/dLCSF gram stain. Pending QuestionHighlights What organism is most likely causing his meningitis? 1 Neisseria meningitidis 2 Staphylococcus epidermidis 3 Staphylococcus aureus 4 Escherichia coli 5 Listeria monocytogenes

1 Neisseria meningitidis The most likely pathogenic organism for bacterial meningitis depends on the age and health of the patient. For this patient's age, Neisseria meningitidis, Streptococcus pneumoniae, and Haemophilus influenzae are all likely causes of meningitis. Neisseria meningitidis is a gram-negative diplococcus. A variety of strains of Neisseria meningitidis exist.

A 32-year-old woman presents several hours after a grand mal seizure. Her husband states that she experienced headache, nausea, vomiting, fever, and "was not herself" a few days before the seizure. She is on glucocorticoid therapy due to a kidney transplant 1 year ago. On examination, you find a lethargic febrile (39°C) dysphasic patient; there is right-sided hemiparesis, and meningeal signs are present, but detailed examination cannot be performed because of lack of cooperation. EEG shows focal abnormalities over temporal lobes, and her CT is normal. A lumbar puncture is performed. Based on your suspected diagnosis, you initiate supportive care, anticonvulsants, and acyclovir. The patient improves over the following days. Question What additional finding would support your suspected diagnosis? 1 Normal CSF: serum glucose ratio 2 Increased CSF protein 3 Extremely elevated opening pressure 4 Increased CSF glucose 5 Predominance of polymorphonuclear cells

1 Normal CSF: serum glucose ratio The patient has signs and symptoms consistent with meningoencephalitis. The etiology of meningoencephalitis includes viral, bacterial, fungal, and tubercular infections. Subarachnoid hemorrhage and mass lesions may also cause meningeal irritation. In an immunocompromised patient with the clinical presentation, as in this case, a viral meningoencephalitis such as herpes should be high on the differential. Its presentation is often atypical: fever, headache, and alteration of consciousness, focal neurological deficits, seizures, and other signs of acute encephalitis. Acyclovir is an antiviral drug with activity against the herpes virus. Lumbar puncture to obtain CSF for evaluation is a key part of determining the etiology of meningitis. CSF characteristics suggestive of viral meningoencephalitis include normal to slightly elevated opening pressure predominance of lymphocytes normal protein level normal glucose level normal CSF: serum glucose ratio (0.6-0.7) Bacterial causes of meningitis are associated with elevated opening pressure, predominance of polymorphonuclear cells (PMNs), elevated protein level, reduced glucose level, and reduced CSF: serum glucose ratio (0.4). Fungal and tubercular causes of meningitis are associated with variable opening pressure, predominance of lymphocytes, elevated protein level, reduced glucose level, and reduced CSF: serum glucose ratio.

An afebrile 76-year-old man has a history of pain and stiffness in his knees bilaterally and in his right hip. The pain is worse after an activity or first thing in the morning. Otherwise, he is healthy and has no other complaints. Upon physical exam, the patient has limited range of motion, joint line tenderness of knees, and mild swelling in the painful joints without warmth. 2 of 18 "tender points" are tender to palpation. A synovial fluid analysis was conducted and it was negative for urate crystals. X-rays of the knees and right hip do not show sclerosis of the bone. Additionally, a Gram stain was conducted and was negative for any bacteria. Question What is the most likely diagnosis? 1 Osteoarthritis 2 Gouty arthritis 3 Fibromyalgia 4 Avascular necrosis 5 Septic arthritis

1 Osteoarthritis This patient presents with knee and hip pain after long use or activity, which is descriptive of osteoarthritis. Additionally, upon physical exam, the patient presents with swelling in his knees and hips, which is due to articular cartilage breaking down. This causes pain and swelling and decreased movement of the joint. In severe OA, muscle wasting can also occur.

A 16-year-old girl presents to the emergency room for evaluation of continuous seizures. The patient has a history of idiopathic epilepsy and takes valproic acid. Two previous EEGs have been normal. This spell occurred at the dinner table after a fight with her mom. Physical exam reveals a well-developed well-nourished young woman lying on a gurney; there is continuous but asynchronous motor jerking of all four extremities. Restraints are in place for patient safety, which causes increased intensity of movements. The head intermittently turns from side to side. Eyes are closed tightly and cannot be passively opened. Deep tendon reflexes cannot be assessed. The response to plantar stimulation is withdrawal bilaterally. Exam otherwise normal. Stat EEG and labs are normal, including CBC with differential, UA on catch specimen, BMP, and arterial blood gas. Chest x-ray is normal. Postictal serum prolactin and creative kinase levels are normal. Question What is the most likely diagnoses? 1 Psychogenic non-epileptic seizure (pseudoseizure) 2 Transient ischemic attack 3 Syncope 4 Panic attack 5 Tonic-clonic seizure

1 Psychogenic non-epileptic seizure (pseudoseizure) Psychogenic non-epileptic seizures (PNES, pseudoseizures) mimic epileptic seizures with tonic-clonic characteristics. Observation, lab work, and electroencephalogram (EEG) can aid diagnoses. Asynchronous movements increased with restraint, preceding stressor, and normal objective testing are characteristic of PNES. During a spell suggesting generalized tonic-clonic seizure, the electroencephalogram should show generalized spike and wave activity; after the spell, it should show generalized slowing and disorganization lasting 30-90 minutes. Even if the active phase of the spell were over before the EEG, if done in this 30- to 90-minute post-event time frame, it could be compared with a later EEG. If an EEG is normal during or immediately after this event, it is major evidence against epilepsy and for PNES. Tonic-clonic seizures consist of a sudden loss of consciousness often lasting <1 minute followed by jerking motions for 2-3 minutes, followed by a stage consisting of flaccid coma.

A 73-year-old man has had a history of pain in that shoulder for over 6 months that kept him from sleeping on the left side; the pain would wake him often. He presents with an inability to actively raise his left non-dominant arm, which began a month ago after his shoulder pain improved. There was no specific injury he can recall, but he felt a pop a month ago while taking out the trash. On inspection, you notice the back of the shoulder appears sunken compared to the other shoulder. Question What is the most likely diagnosis? 1 Rotator cuff tear 2 Impingement syndrome 3 Adhesive capsulitis 4 Glenohumeral dislocation 5 Proximal humerus fracture

1 Rotator cuff tear

A 68-year-old man presents due to lower back pain. The patient reports the pain has been present for the past 3 months and seems to be getting worse. Upon physical exam, the patient appears to get relief of pain when bending forward. Upon standing and extension of the lumbar spine, the patient reports pain. He denies decreased range of motion in the shoulders, neck, and hips. The spine is not in an S or C shape. T2 weighted imaging shows disc degeneration. X-rays show symmetrical joint spaces. C-reactive protein (<1.0 mg/L) and ESR (<40 mm/hr) are in normal range. Gram stain is negative for Staphylococcus aureus. Question What is the most likely diagnosis? 1 Spinal stenosis 2 Osteoarthritis 3 Polymyalgia rheumatica 4 Scoliosis 5 Septic arthritis

1 Spinal stenosis Spinal stenosis involves a decrease in pain when leaning forward and worsened pain from standing and extension of the lumbar spine. The T2 weighted imaging shows disc degeneration, which can lead to narrowing of the spinal canal. Hypertrophic facet joints can also be seen on X-ray in patients with spinal stenosis.

A 32-year-old woman presents with a 3-month history of right hand pain and paresthesia of the thumb, index, and middle fingers. She works in a retail sales office and spends much of the day typing. Question What associated physical finding may indicate an advanced case of this suspected diagnosis? 1 Thenar atrophy 2 Hypothenar atrophy 3 Weakness on extension of the DIP joints 4 Decreased gap to 2 mm on 2-point discrimination test 5 Weak radial artery pulse

1 Thenar atrophy

A 19-year-old woman presents with worsening headaches. She reports a multi-year history of episodic throbbing headaches. They have intensified, and she now misses classes and work periodically as a result. The headaches occur about 4-6 times per month recently, up from 1-2 per month when she first started experiencing them. Her headaches last 2-3 days and are accompanied by nausea, vomiting, and light sensitivity. After the headache resolves, she denies any residual symptoms. She denies neurologic symptoms, such as vision or taste changes, gait disturbances, and memory loss. She has tried multiple over-the-counter pain medications without relief. Her mother and maternal aunt experienced similar headaches. She reports some increased stressors and less sleep since recently starting college. Her past medical history is unremarkable, with no known medical conditions (except for the headaches), no surgeries, no chronic medications, and no drug allergies. She has never been sexually active and reports regular menses. She denies the use of tobacco, alcohol, and recreational drugs. On physical exam, the patient appears comfortable and reports no headache at this time. Her entire exam, including neurological, is normal. Question What test result best supports the most likely diagnosis? 1 Inflammation on temporal artery biopsy 2 Normal magnetic resonance imaging (MRI) 3 Slow wave pattern on electroencephalogram (EEG) 4 Positive toxicology for cocaine 5 Thickened sinus mucosa on computed tomography (CT)

2 Normal magnetic resonance imaging (MRI)

A 37-year-old woman presents with a history of right-sided facial paralysis and periauricular discomfort since she awoke this morning. She is afebrile. She had a "cold sore" 1 week prior to her symptoms, but this resolved without complications. During her neurologic exam, she was discovered to have an inability to raise her right eyebrow and close her right eye completely. She also has drooping of the right corner of her mouth. The rest of her neurologic exam is normal. There are no masses or rashes evident. She denies history of prior CVA or neurologic illnesses. She does not take any medications. Question What is the most likely diagnosis? 1 Trigeminal neuralgia 2 Bell's palsy 3 Multiple sclerosis 4 Myasthenia gravis 5 Primary lateral sclerosis

2 Bell's palsy

A 15-month-old toddler presents with sudden onset of generalized tonic and clonic convulsions for the last 30 minutes. Parents report that the patient was irritable in the minutes preceding the events, but there is no history of trauma, fever, or vomiting before the onset of convulsions. This is the first episode of seizure, and there is no history of convulsions in the family but the father has a history of cerebral aneurysm requiring surgical clipping. The patient's birth history, neonatal period, and developmental milestones are normal. Question After initial stabilization by securing the airway, oxygenation, and controlling the acute convulsion, what is the best investigation for this patient? 1 Cerebrospinal fluid examination 2 CT scan of the head 3 Electroencephalogram 4 Cranial ultrasonography 5 MRI scan of the head

2 CT scan of the head CT scan of the head is the best diagnostic imaging study in a previously asymptomatic child who presents with status epilepticus, particularly if there is suspicion of intracranial hemorrhage, a space-occupying lesion, or midline shift. It is a non-invasive, rapid, widely available procedure. It has a short imaging time and requires less sedation in young children. CT imaging is largely replaced by MRI, but it remains the most sensitive modality to detect intracranial hemorrhage and calcifications during the first 24 hours.

A 35-year-old man presents with recurrent headaches. They occur on one side of his head and feel like sharp stabbing pain just above his left eye. He notes eye redness and watering with the headaches. Each headache lasts for 2 hours, and they have been occurring daily for 2-3 weeks. He reports a similar pattern of headaches last year. He denies any known family history of headaches similar to his or migraines. Physical examination is within normal limits. Question What is the most likely diagnosis? 1 Tension headache 2 Cluster headache 3 Cerebral aneurysm 4 Hemicrania continua 5 Classic migraine

2 Cluster headache

A 35-year-old man presents with headaches; he describes them as occurring on one side of his head with a sensation of a sharp stabbing pain just above his left eye. The episodes last for 2 hours, have occurred daily for 2-3 weeks, and have been presenting in this pattern every 3-4 months for the past 2 years. The patient describes his nose as feeling congested during these periods; his left eye also appears red. Aggravating factors appear to be alcohol and stressful situations. He denies any family history of migraines or headaches similar to his. Diagnostic and laboratory studies are pending. Question What is the most likely diagnosis? 1 Tension headaches 2 Cluster headaches 3 Cerebral aneurysm 4 Hemicrania continua 5 Classic migraine

2 Cluster headaches

A 13-year-old boy presents with pain in his right leg, present for about 2 months but worsening over time. He has developed a low-grade fever. He denies any known injury to the area. On examination, there is tenderness and mild swelling near the right fibula. X-ray reveals an invasive lesion involving the right fibula, with a periosteal onion-skin reaction. ]Question What is the most likely diagnosis? 1 Plasmacytoma 2 Ewing's sarcoma 3 Multiple myeloma 4 Endochondroma 5 Osteochondroma

2 Ewing's sarcoma Ewing's sarcoma is a neoplastic tumor of the bone that most commonly affects the lower extremities, including the long bones and pelvic bones. Patients typically present with swelling and pain. Classic X-ray findings show a destructive lesion described as "moth-eaten" or periosteal onion-skin reaction.

A 65-year-old female patient presents with pain and morning stiffness in the knees and right toes. She was an athlete in school and college days. She has not been able to go on her daily morning walks for 3 months. Pain is worse in the evenings and improves with rest. There is no history of trauma or fractures. Question What additional changes are associated with this condition? 1 Fever, fatigue, weight loss 2 Heberden's nodes 3 Inflammatory changes in the joint fluid aspirate 4 Redness and heat over the affected joints 5 Swan neck deformity

2 Heberden's nodes This patient most likely has osteoarthritis. It is a common cause of joint disease in older adults, associated with decreased movement and increased stiffness. There are hard bony swellings (Heberden's nodes) found in the distal interphalangeal joints Joints have a layer of cartilage that protects the bones. In osteoarthritis, this cartilage layer gradually degrades and the bones in the joints come in direct contact with each other. This pathological process damages the bones due to the constant contact, which in turn leads to the development of Heberden's nodes.

A 36-year-old woman presents with fatigue and diplopia; symptoms started in the morning and worsened during her time at work. On examination, the patient has ptosis bilaterally and decreased ocular muscle power. A CT scan of the chest shows a thymoma. Question What is the pathophysiology of this patient's neuromuscular disorder? 1 Defective plasma cholinesterase 2 Immune-mediated destruction of the acetylcholine receptor 3 Deficient quantity of plasma cholinesterase 4 Impaired repolarization at the motor endplate 5 Deficiency in dystrophin

2 Immune-mediated destruction of the acetylcholine receptor

A 16-year-old boy presents following the striking of a wooden door with a closed fist an hour ago when he was angry at his mother. He is neurovascularly intact, and the skin is closed. There is an obvious deformity with a loss of small finger metacarpal knuckle. Radiographs reveal an oblique mid-shaft fracture of the fifth metacarpal with a palmar angulation of 45°. Question What physical exam technique must be performed to check for rotation of the fracture? 1 Look at the fingernails while fingers are extended for position in relationship with each other. 2 Look for finger malposition when the fingers are flexed into the palm. 3 Look for folds in the skin over the metacarpal head. 4 Check for motion of the PIP and DIP joints. 5 Look for palmar edema of the affected digit metacarpal.

2 Look for finger malposition when the fingers are flexed into the palm. With a fractured fifth metacarpal, there is an increased incidence of malrotation of the distal digit due to the potential loss of metacarpal height and lack of ligamentous support of the metacarpal head by the intermetacarpal ligaments. Therefore, rotation must be checked closely to ensure proper position of the distal fracture segment in relation to the proximal fracture fragment for functional healing to take place. This is best accomplished by asking the patient to make a partial fist (gently) and watching for finger malposition (normally all the digits of a closed fist point towards a single spot on the scaphoid) or "cross-over." Be sure to compare the injured hand to the uninjured one as some "normal cross-over" can be seen with the fifth metacarpal.

A 19-year-old male patient presents with pain in the right dominant shoulder, which was injured when he was blocked while shooting a basketball. Physical examination includes significant pain and resistance to passive arm movement. The arm is slightly abducted and externally rotated to minimize pain. Question What finding is expected on physical exam? 1 Tenderness to palpation directly over the acromioclavicular joint 2 Loss of surface contour of the shoulder 3 Full passive range of motion 4 Ecchymosis over the clavicle extending to the shoulder 5 Full active motion of the shoulder

2 Loss of surface contour of the shoulder This patient's mechanism of injury and physical examination are consistent with a shoulder dislocation. In dislocation of the glenohumeral joint (anterior dislocation of the shoulder), the arm is slightly abducted and externally rotated. With the dislocation of the glenohumeral joint, there is a loss of normal contour (rounded appearance) of the shoulder.

A 75-year-old man presents to his pain management office for follow-up after a lumbar laminectomy 2 weeks ago. He states that he has had a fever for the past 6 days. He denies any headache, chest pain, shortness of breath, cough, abdominal pain, nausea, diarrhea, vomiting, or extremity pain. He has a past medical history of hypertension, hyperlipidemia, chronic lumbar stenosis (now status post laminectomy), and type 2 diabetes mellitus. He also has a history of intravenous drug use. He was referred for an MRI of the spine, which revealed destruction of the vertebral body of L4 with significant collapse. Question What is the most likely additional clinical manifestation in this patient? 1 Foul urinary odor 2 Lower back pain 3 Lower extremity sensory loss 4 Presence of a sinus tract 5 Bowel incontinence

2 Lower back pain This patient is experiencing acute osteomyelitis secondary to the recent lumbar laminectomy. His risk factors include his recent surgery, history of IV drug use, and diabetes mellitus. Most cases are monomicrobial in etiology. Of episodes of acute vertebral osteomyelitis, 40-50% are caused by Staphylococcus aureus. The signs and symptoms of vertebral osteomyelitis are non-specific. Only about 50% of patients develop fever >38°C (100.4°F), perhaps because analgesic drugs are frequently used by these patients. Lower back pain is the leading initial symptom (>85% of cases). The location of the pain corresponds to the site of infection: the cervical spine in ~10% of cases, the thoracic spine in 30%, and the lumbar spine in 60%. Local pain, swelling, erythema, and edema, scars, local disturbance of wound healing, and the cardinal signs of inflammation may all be observed on physical exam. If periosteal tissues are involved, point tenderness may be present.

A 5-month-old male infant is brought to the emergency department after a possible seizure episode. His mother states he was born full-term without any complications, and he was well until 2 days ago when he developed a fever. He vomited multiple times yesterday and was irritable. He has not had diarrhea or a cough. He was given antipyretic medication for his fever. He has no known allergies. His immunizations are current. His developmental milestones have been in accordance with his age. On physical exam, temperature is 102.7°F, pulse is 154/min, BP is 90/50 mm Hg, RR is 20/min. He is lethargic and pale; there are no focal neurological deficits or active seizure activity. Non-contrast CT of the head is unremarkable. Question After drawing blood samples for investigations, what is the most appropriate next step? 1. Ultrasound of the head 2 Lumbar puncture 3 Initiation of empiric antibiotics 4 MRI of the head 5 CT of the spine

2 Lumbar puncture This patient's presentation is consistent with acute bacterial meningitis.

A 33-year-old woman presents with tingling and numbness in the palms, thumbs, and index fingers bilaterally. Her symptoms are worse during the night. Recently, she noticed that she has difficulty grasping small objects. She works at a poultry farm and is involved in assembly packing. On examination, Tinel's sign and Phalen's test are positive. Question What nerve is compressed? 1 Ulnar nerve 2 Median nerve 3 Radial nerve 4 Musculocutaneous nerve 5 Posterior interosseous nerve

2 Median nerve

A 65-year-old woman presents with general weakness for the past few days; it gets worse as the day progresses. She says she has been having double vision, and you notice she has poor posture. A blood test reveals the presence of antibodies to acetylcholine receptor. A repetitive nerve stimulation test showed a decremental response. Question What is the most likely diagnosis? 1 Marfan syndrome 2 Myasthenia gravis 3 Duchenne muscular dystrophy 4 Rheumatoid arthritis 5 Systemic lupus erythematosus

2 Myasthenia gravis

A 23-year-old woman presents with increasing fatigability and muscle weakness. On further questioning, she reveals that she has experienced difficulty swallowing over the past month, and there have been episodes of double vision as the day progresses. Several of her professors have expressed concern because she looks sleepy during the day. A CT of the chest reveals an anterior mediastinal mass. Question What is the most likely diagnosis? 1 Myasthenic syndrome 2 Myasthenia gravis 3 Amyotrophic lateral sclerosis 4 Botulism 5 Hyperthyroidism

2 Myasthenia gravis Myasthenia gravis is an autoimmune disorder with cardinal symptoms of weakness and fatigue, more common among young women. Clinical features include ptosis, diplopia, chewing and swallowing difficulties, respiratory difficulties, and asymmetrical proximal limb weakness with normal deep tendon reflexes. Initial symptoms are often related to muscles controlled by cranial nerves, which are affected first. Investigations done in patients with myasthenia gravis include: Edrophonium test Acetylcholine receptor antibody test CT scan of the thorax to detect a thymoma if present Electromyogram (EMG) This patient has a mediastinal mass consistent with a thymoma. Removal of the thymus (thymectomy) will produce improvement in >85% of patients and complete remission in some. Presence of a thymoma is an indication for surgery.

A 34-year-old woman presents Monday morning with knee pain. She states that she was playing with her two children in their backyard over the weekend and fell onto solid ground, landing directly on her knees. She noted immediate and significant right knee pain. She notes significant swelling; it accompanies the knee pain, which she rates as an 8/10 on a numerical pain scale. Physical examination reveals significant obvious joint effusion and exquisite focal tenderness to palpation over the patellar area of the right knee; the left knee has no obvious abnormalities. Results of the anterior drawer, McMurray, and varus/valgus stress testing are within normal limits. Question Given the history and physical examination findings, what is the most likely diagnosis? 1 Bone cyst 2 Patellar fracture 3 Ligament tear 4 Tibial fracture 5 Femoral shaft fracture

2 Patellar fracture

arents bring their 4-year-old daughter in because of knock-knee. She is otherwise healthy, and her height is in the 50th percentile for age. On examination, she has about 10 degrees of valgus. Question What should you recommend? 1 X-rays of the lower extremities 2 Reassurance that this is normal for age 3 Orthopedic shoes 4 Long leg braces 5 Lab tests for markers of bone turnover

2 Reassurance that this is normal for age

A 33-year-old woman presents with joint pain and morning stiffness that lasts more than an hour. Her past medical history is significant for postpartum depression 2 years ago. Her BMI is 18. Physical examination is remarkable for bilateral pain and swelling of her metacarpophalangeal (MCP) joints and ulnar deviation. Laboratory evaluation reveals positive antibodies to citrullinated peptide. Question Based on the most likely diagnosis, what is the primary risk factor in this patient's case? 1 Age 2 Sex 3 Depression 4 BMI 5 Pregnancy

2 Sex

A 56-year-old man wakes up in the morning to find that he has a swollen, red, and painful big toe on his left foot. He had been on a cruise to the Bahamas 2 days earlier, and he spent much of the time eating and drinking. He normally has 1 glass of wine with dinner on the weekends, but his alcohol consumption increased substantially while on the cruise. He also did a great deal of walking in an attempt to make up for his excesses. He goes into his physician's office, and tests are run. An X-ray shows no acute fracture, and his vital signs are within normal limits. Blood work shows an increase in uric acid, but it is otherwise normal. He begins treatment and feels better within 24 hours. Question What most accurately describes the typical presentation of this disease? 1 Polyarticular arthritis 2 Small lower extremity joints 3 Caucasians more likely than African Americans 4 Women more likely than men 5 Under age 25

2 Small lower extremity joints

A 23-year-old woman presents with a rash and swollen joints. She had been healthy previously, and the only medication she takes is acetaminophen. A review of systems includes recent fevers and a 5 lb weight loss in the past month; she is also experiencing photosensitivity and hematuria. She denies oral ulcers, nasal congestion/discharge, ear pain, pleuritic symptoms, chest pain, neural symptoms, bruising, and bleeding. On physical exam, her temperature is 101°F; her blood pressure is 130/85 mm Hg. She has a malar rash as well as diffusely swollen, warm, and tender joints. Her cardiopulmonary exam is normal. She has no costovertebral angle tenderness. Trace bipedal edema is noted. Diagnostic studies reveal: Erythrocyte sedimentation rate 50 mm/hour Hemoglobin 11 g/dL Anti-double-stranded DNA antibodies Positive Anti-nuclear antibody Positive Anti-histone antibody Positive Platelet count 200,000 cells/mL White blood cell count 12,000 cells/mm3 Chest X-ray Normal Urinalysis --> No bacterial growth. Numerous dysmorphic red blood cells. 10 white blood cells/high power field. Red blood cell casts. 510 mg albumin/g creatinine. Complement 3, 4 Both reduced Anti-neutrophil cytoplasmic antibody Negative Question What is the most likely diagnosis? 1 Drug-induced lupus 2 Systemic lupus erythematosus 3 Granulomatosis with polyangiitis 4 Microscopic polyangiitis 5 Henoch-Schönlein purpura

2 Systemic lupus erythematosus This woman most likely has systemic lupus erythematosus (SLE), given her symptoms of hematuria, photosensitivity, and arthritis, in addition to the findings of urinary red cell casts, proteinuria, and hypocomplementemia. The most common clinical features included arthritis, hematological abnormalities, malar rash, photosensitivity, renal involvement, oral ulcers, serositis and neurological involvement. Renal involvement is frequent in SLE, and it may include a variety of glomerular diseases. Hypocomplementemia may occur due to complement consumption in the glomeruli. Focal proliferative and diffusely proliferative glomerulonephritis are two forms of glomerulonephritis that might be seen in this patient if biopsied. If proven, she might be treated with immunosuppressive therapy, such as prednisone and cyclophosphamide. Patients with SLE nephritis may recover, relapse, progress to a different form of lupus nephritis, or progress to end-stage renal disease. In a classification scheme revised by the Systemic Lupus International Classification Clinic (SLICC) in 2012, patients who have "4 SLICC criteria including at least one immunological criterion (e.g., elevated ANA) and at least one clinical criteria (e.g.,malar rash) or biopsy-proven nephritis compatible with systemic lupus in the present of either ANA or anti-dsDNA" can be classified as having systemic lupus erythematosus. The ACR criteria are also used with diagnosis being made when 4 out of 11 criteria are met.

A 40-year-old African American woman presents with diffuse headache and joint pain. The headache started few days ago; it is dull and becoming progressively worse. Joint pain is localized in fingers, starts in the morning, and improves during the day; it returns when she gets tired. 2 months ago, she was treated in the ED because of several weeks of lasting fatigue, low-grade fevers, joint pain, hair loss, and oral ulcers. Her laboratory tests were normal, except for positive VDRL and antinuclear antibody tests results. Urine and blood cultures were negative for evidence of infection, and her chest X-ray was normal. Because she felt better after a 10-day tapering course of prednisone, she did not follow up with a healthcare provider at that time. She did not appear to the scheduled control. Today, your examination reveals an ill-appearing woman in distress. Her temperature is 39°C. Her fingers are swollen and red, and she has a malar rash and oral ulcers. Question What additional test should you order as the most specific for her condition? 1 Synovial fluid analysis 2 Urine protein/creatinine ratio 3 Antibodies to the Sm antigen 4 Cerebrospinal fluid analysis 5 Complement system test

3 Antibodies to the Sm antigen The diagnosis of SLE is based on clinical features and the presence of autoantibodies. Current criteria for the diagnosis are ≥4 of the following: malar rash, discoid rash, photosensitivity, oral ulcers, arthritis, serositis, renal disorder, neurologic disorder, immunologic disorder, antinuclear antibodies This patient meets the criteria. She has a history of positive ANA and alopecia; there is the presence of a neuropsychiatric disorder, oral ulcers, arthritis, and fever, with infection ruled out. Antibodies to the Sm antigen in the presence of characteristic clinical picture are the most specific for SLE. VDRL is often false positive in SLE patients (in about 1/4 of patients). Clinically, VDRL-positive patients present differently than VDRL negative.

A 40-year-old woman presents occasional constipation that she relates to her diet. She is married with two sons. Aside from mild anxiety, she is dealing appropriately with her duties as a librarian. She does not smoke, she consumes alcohol only recreationally, and she weighs 187 lb. She is well-nourished and communicates well. Lung, cardiac, abdominal, and neurological examinations are unremarkable. Thyroid is normal in size and consistency. Menses are normal, and drug history is negative except for "hormone pills" for contraception. Question What imaging modality would you recommend for osteoporosis screening and when should it be done? 1 Bone densitometry after menopause 2 Hip and spinal X-ray after menopause 3 Bone densitometry at age 65 4 Hip and spinal X-ray at age 65 5 Bone densitometry at age 55

3 Bone densitometry at age 65

A 53-year-old man presents with a 2-week history of severe headaches that occur primarily at night. The patient is pacing while he is talking. The pain surrounds one eye and lasts for 30-90 minutes. He also states that there is ipsilateral lacrimation, conjunctival injection, and nasal congestion during the attacks. The patient states that he has had these headaches once a day over the last week. He cannot point to any one thing that causes them. On examination, the patient has features of partial Horner's syndrome. His vital signs are temperature: 97.0°F, heart rate: 80/min, respiration 16/min, and blood pressure: 126/80 mm Hg. Question What is the most likely diagnosis? 1 Common migraine 2 Classic migraine 3 Cluster headache 4 Stress headache 5 Chronic daily headache

3 Cluster headache

A 12-month-old girl presents with her parents after a 3-day history of intermittent episodes of strange behavior. A neurologist is consulted because the parents are concerned she is having some form of seizure activity. Her health history includes 2 episodes of otitis media, but she is otherwise healthy. Her initial vital signs and physical exam by the emergency room staff are all normal. Question What description of the infant's strange behavior would lead the neurologist to suspect a diagnosis of simple partial seizures? 1 Facial and right arm twitching lasting 1 minute, then loss of consciousness 2 Blank stare and impaired awareness for 10-20 seconds 3 Eye deviation with facial twitching that lasts 1-2 minutes 4 Loss of consciousness, then stiffening of the body and extremities 5 Brief periods of whole body going limp, then a brief loss of consciousness

3 Eye deviation with facial twitching that lasts 1-2 minutes

A 19-year-old man presents with pain and deformity of his right dominant shoulder after a sudden jerking movement to that shoulder from a wrestling competitor approximately 1 hour ago. He was unable to continue wrestling and has pain with any movement of the right shoulder. On exam, you see a loss of normal shoulder contour anteriorly. There is no focal joint or bony tenderness. Question What is the most likely diagnosis? 1 Rotator cuff tear 2 Shoulder separation 3 Glenohumeral dislocation 4 Fractured clavicle 5 Sternoclavicular subluxation

3 Glenohumeral dislocation

A 12-year-old girl presents with a 3-day history of progressive weakness and paresthesias in lower legs. Yesterday, she developed weakness in both upper extremities. She is unable to walk without assistance, so her mother brought her in. She had a sore throat 2 weeks ago. PMH is significant for measles and mumps. Because of religious beliefs, she has not had any immunizations. Physical examination reveals a well-developed well-nourished girl. She is awake, alert, and in no acute distress. Oral temperature 98.7°F, blood pressure 140/80 mm Hg, heart rate 84/min and regular, respirations 22/min and unlabored. Speech is moderately dysarthric. She can smile weakly but cannot raise her eyebrows against resistance. Pupillary responses are normal. There is mild upper extremity and severe lower extremity weakness, greater distally than proximally. Reflexes are hypoactive-to-absent. Sensation is intact, except for mildly impaired position and vibratory sensation in both feet. Question What is the most likely diagnosis? 1 Tetanus 2 Diphtheria 3 Guillain-Barré syndrome 4 Botulism 5 Myasthenia gravis

3 Guillain-Barré syndrome

A 32-year-old man presents with a 2-day history of high fever and progressive severe headaches that are associated with blurred vision and increasing confusion. The patient is normally healthy, and he does not have a remarkable past medical history. He is married. He does not drink alcohol, smoke, or use recreational drugs. He has not had any blood transfusions and takes no medications. On physical examination, he appears ill and disorientated to time, place, and person. His pulse is 110 bpm, temperature 39°C (102°F), respirations 22/minute, and blood pressure 115/70 mm Hg. He is well-hydrated. He has no scleral icterus, pupils are equal and reactive, and fundi are normal. His cranial nerve examination is intact. He does not respond to pain stimuli appropriately. He can move all of his limbs. Deep tendon reflexes are normal; plantar reflexes are equivocal. His neck is supple and there is no palpable adenopathy. Skin exam is normal without rashes. His lab work includes: white blood cell count (WBC) = 18x109/L with 70% polymorphonuclear neutrophils and 30% lymphocytes. erythrocyte sedimentation rate = 90 mm/hour WBC = 18x109/L CO = 228 mg/dL. Total bilirubin = 1 mg/dL PMN neutrophil = 70%. Cl = 102 mEq/L. Alkaline phosphatase = 112U/L lymphocyte = 30%. BUN=29 mg/dL. AST=22 U/L. Hgb=13.1 g/dL Cr =1mg/dL. ALT=26 U/L. Hct=39%. Glucose=101 mg/dL. Amylase=477 U/L. Plt=200x109/L. Calcium=9.2 mg/dL. Lipase=625 U/L Na=138 mEq/L Total protein=6 g/dL LDH=300K4 mEq/L Albumin=3 g/dL ESR=90 mm/hour Results of drug screening are negative and his urinalysis is normal. The lumbar puncture results are as follows: Cerebrospinal pressure --> Increased Protein level --> 180 mg/dL Glucose --> 92 mg/dL White blood cell count --> 116 per microliter (all lymphocytes) Red blood cell count --->. 80 per microliter Gram-staining results --> White blood cells with no organisms Oligoclonal bands ---> Absent IgG index --. Normal Question An MRI scan of the brain demonstrates gadolinium-enhancing lesions in the temporal region. What is the most likely diagnosis? 1 Acute multiple sclerosis 2 Central nervous system lymphoma 3 Herpes simplex virus encephalitis 4 Multifocal leukoencephalopathy 5 Viral meningitis

3 Herpes simplex virus encephalitis This patient has a fever, severe headache, altered mental status, and abnormal levels of lymphocytes in the CSF. These findings, along with the results of the MRI scan, suggest herpes simplex virus (HSV) encephalitis. Temporal lesions that enhance with gadolinium infusion are typical. HSV encephalitis occurs in 10% of all cases of encephalitis in the US. There is a high mortality rate and morbidity rate. Herpes simplex type 1 is the most common cause and type 2 causes neonatal encephalitis (acquired by mother). The illness typically occurs over several days. The CSF findings in this patient typify the condition. Specifically, the CSF pressure is increased with pleocytosis. The RBCs reflect the hemorrhagic nature of the CNS lesions. The protein level is increased; the glucose level may be normal or decreased.

A 15-year-old girl presents with loss of consciousness. She is accompanied by her mother, who states that the patient initially fell ill several days ago with a headache, muscle aches, and fever. The patient developed a severe headache today, accompanied by double vision, difficulty speaking, confusion, and eventual loss of consciousness. She has not taken any medications aside from acetaminophen (Tylenol) for her fever. Her mother states that her daughter is usually active and had been playing soccer regularly until she became ill; the patient has been fairly healthy aside from occasional cold sores. Past medical history is significant for frequent ear infections as a toddler that were treated with tympanostomy tube placement at age 2. Brain imaging reveals edema of the temporal lobe. Question What component of the patient's history is most likely contributing to her current illness? 1 Recent acetaminophen use 2 Playing soccer regularly 3 History of cold sores 4 History of frequent ear infections as a child 5 History of tympanostomy tube placement

3 History of cold sores The patient is exhibiting neurological symptoms and imaging results consistent with encephalitis. Encephalitis is typically viral in nature, with herpes simplex virus being the most common cause. Cold sores are associated with herpes simplex virus.

A 4-month-old female infant is presented for a well-child checkup. She was a spontaneous vaginal delivery at 39.5 weeks without complications. The mother notes she has been irritable and has not been eating well. No cough or fever. No one smokes at home. On exam, you note impaired extraocular movements, especially in the upward gaze, and a bulging anterior fontanel. There is increased tone of the legs. Skin exam is normal. Like her last visit, the length and weight are 50th percentile. Head circumference was formerly at the 75th percentile and is now above the 99th percentile. Question What is the most likely diagnosis? 1 Positional plagiocephaly 2 Craniosynostosis 3 Hydrocephalus 4 Neurofibromatosis 5 Catch-up growth

3 Hydrocephalus

A 17-year-old male college freshman presents with fatigue, back pain, and stiffness. The pain has been present for the past several months, but it appears to be worsening. The back symptoms are worse at night and first thing in the morning; they improve somewhat during the day. He reports the pain improves with exercise. He is having difficulty staying productive at school because he is always tired. Question What laboratory finding is most probable? 1 Decreased c-reactive protein 2 Increased b-natriuretic protein 3 Increased erythrocyte sedimentation rate 4 Decreased alkaline phosphatase 5 Increased uric acid level

3 Increased erythrocyte sedimentation rate This patient presents with signs and symptoms indicative of inflammatory arthritis, most likely ankylosing spondylitis (AS). Characteristic symptoms include back pain that is worst in the morning and improves with exercise. Back stiffness and extreme fatigue are also reported. AS is more common in male patients. As with other inflammatory arthritides, patients usually present at a younger age (18-35) relative to patients with mechanical causes of back pain or arthritis. AS is a clinical diagnosis, but many patients will have elevated erythrocyte sedimentation rates and c-reactive protein rates, both of which are non-specific markers of inflammation. Approximately half of patients will have an increased alkaline phosphatase level. HLA-B27 test may also be completed in the workup.

You are evaluating a 34-year-old African American man for a 5-week history of increasing right groin pain. He denies any injury or history of similar pain. The pain is worse with movement and has progressed to the point that the patient has severe pain with bearing weight. He denies fever, chills, urinary symptoms, or any other issue at this time. He has a past medical history of sickle cell disease and hypothyroidism. Physical exam reveals tenderness upon palpation of the groin with increased pain on both active and passive range of motion of the hip. Homan's sign is negative. Distal pulses and sensation are intact and normal. Question What diagnostics study should be ordered at this time? 1 Ultrasound of the right lower leg 2 CBC 3 MRI of the right hip with and without contrast 4 CXR 5 Urinalysis

3 MRI of the right hip with and without contrast This patient's presentation is concerning for avascular necrosis of the right hip. The most appropriate diagnostic test to order at this time is an MRI of the hip with and without contrast. Avascular necrosis (osteonecrosis) is a focal infarct of the bone due to an interruption of blood supply. Avascular necrosis (AVN) most commonly occurs in the hips, jaw, and scaphoid bone of the hand. Risk factors for AVN include trauma, coagulopathy, alcoholism, chronic corticosteroid use, sickle cell disease, and autoimmune diseases such as RA and SLE.

A 22-year-old woman presents with headache. The pain is located on the right side of her head and is described as throbbing. The headache has lasted for approximately 10 hours and has been unrelieved by acetaminophen and ibuprofen. The pain worsens with movement, exposure to light, and loud noises. Neurological examination is within normal limits. Question What is the most likely diagnosis? 1 Tension headache 2 Cluster headache 3 Migraine headache 4 Brain tumor 5 Pseudotumor cerebri

3 Migraine headache

A 14-year old girl presents due to right leg pain. She is a competitive gymnast and works out in the gym 6 days a week. Upon physical exam, there is knee pain, swelling, and tenderness at the tibial tubercle. The patient's muscles are very tight in front and behind her thigh. All laboratory findings are within normal limits. Question What is the most likely diagnosis? 1 Iliotibial band syndrome 2 Kneecap dislocation 3 Osgood-Schlatter disease 4 Patellar tendinitis 5 Sinding-Larsen-Johansson syndrome

3 Osgood-Schlatter disease Osgood-Schlatter disease usually affects children going through a growth spurt and who are involved in jumping-centric sports. The patient presents with the appropriate symptoms of pain, swelling, and tenderness around the tibial tubercle.

An 89-year-old Caucasian man is brought in by his daughter due to pain in his left shin. He bumped his leg on a coffee table about 3 weeks ago; he developed some mild discomfort, bruising, and a small gash in the skin. It seemed as if he was healing well, but his condition has worsened over the past few days. Now he is moaning due to pain, and he says it hurts to walk on the leg. He describes the pain as "horrible" and an 8/10. The daughter reports the pain keeps him up at night and is unresponsive to ibuprofen and narcotic pain pills. He denies fever and chills. On physical exam, the patient is in obvious pain and is assisted to the exam table with limited weight bearing on his left leg. The lower extremities are examined; significant findings include healing and a scabbed lesion of approximately 3 cm in length across mid-tibia, with surrounding erythema and edema. Tenderness is elicited along the shin, extending well past the area of erythema. Homan's sign is negative. Distal pulses, temperature, coloration, knee range of motion, and lower extremity reflexes are symmetric and normal. Right lower extremity is normal. Question What is the most likely diagnosis? 1 Claudication 2 Gout 3 Osteomyelitis 4 Pes anserine bursitis 5 Stress fracture

3 Osteomyelitis This patient presents with a likely osteomyelitis: infection of the bone tissue. In this patient's case, some bacterial spread from a relatively minor injury at the skin surface is likely responsible. Osteomyelitis can affect all age groups, but it can especially affect the elderly population and those with poor peripheral circulation. Long bones and vertebrae are most commonly affected. This patient's pain is out of proportion with exam findings. With acute osteomyelitis in adults, exam findings are often limited to local erythema and edema (with tenderness), but with exaggerated pain.

An 18-year-old man with no significant past medical history presents with pain and swelling over the upper right knee for 5 months. The pain was initially insidious, dull, and achy. Over the past several weeks, however, it has gradually become progressively more severe and unremitting, often waking the patient at night. He also notes increased swelling, warmth, and erythema. He denies a history of injuries, accidents, trauma, surgeries, or sexual encounters. His physical exam reveals a noticeable limp, reduced right knee range of motion, and localized tenderness and swelling to the distal anterior femur. Question What is the next step in the management of this patient? 1 Begin physical therapy 3 times a week for 6 weeks. 2 Begin a xanthine oxidase inhibitor and indomethacin. 3 Perform a right knee radiograph. 4 Prescribe a trial of ibuprofen and observe for 2 weeks. 5 Refer the patient for a bone scintigraphy scan.

3 Perform a right knee radiograph. This patient's presentation is suspicious for the malignant bone tumor known as osteosarcoma. The imaging of any bone tumor should begin with a radiograph of the involved area. Such an X-ray image is often helpful in the diagnosis of bone sarcomas; osteosarcoma often has a sunburst appearance of calcification on X-ray imaging, which is virtually diagnostic. Once the diagnosis has been established, a chest X-ray, CT scan of the chest, and bone scan should be performed to evaluate for metastatic disease. As with soft tissue sarcomas, the most common site of metastasis from bone sarcoma is the lungs, warranting a chest X-ray and CT chest.

A parent brings in their 6-year-old child whose teacher is concerned about episodes of abnormal behavior at school. The teacher has noticed the patient "staring off into space" frequently throughout the day; the teacher cannot get the patient's attention while staring. After the end of the "staring episode," the patient continues to behave as if nothing happened and they are not aware of their own behavior. The parent states that there are times when the patient does not seem to be paying attention to what the parent is saying. Shortly after these episodes, the patient engages in conversation without any problem, so the mother did not think the episodes were an issue. There is no concern about other abnormal behavior or discipline issues at home or school. Question What is the most likely diagnosis? 1 Attention-deficit disorder (ADD) 2 Syncopal episodes 3 Petit mal (absence) seizures 4 Narcolepsy 5 Childhood disintegration disorder (CDD)

3 Petit mal (absence) seizures

A 17-year-old male high school football player presents after being tackled and slammed onto his right dominant shoulder forcefully 2 hours ago during a game. He had immediate pain but was able to continue playing. He has full active and passive range of motion, but some pain (6/10) with abduction. There is no obvious deformity, and the skin over the shoulder is intact and not tented. Question What initial radiographic procedure should be ordered? 1 Ultrasound 2 CT without contrast 3 Plain film radiograph 4 MRI without contrast 5 MR arthrography

3 Plain film radiograph

A 54-year-old male patient presents with a 2-month history of left non-dominant shoulder pain. There is no history of trauma, but the pain began about a week after shoveling wet heavy snow from a 100-foot driveway. At first, the pain seemed to come and go from day to day, but it has progressively worsened and has become more constant. Pain is worse with overhead use. He cannot sleep on the left side and will wake up if he rolls over onto the left shoulder. Drop arm test is negative. Question Considering the most likely diagnosis, what other exam finding is expected? 1 Tenderness at the acromioclavicular joint 2 Restricted active and passive motion of the shoulder 3 Positive Neer and Hawkins signs 4 Positive sulcus sign 5 Edema of the shoulder muscles

3 Positive Neer and Hawkins signs Drop arm test is positive in rotator cuff tears. With the other symptoms, it is likely this patient has impingement syndrome. Neer and Hawkins signs are positive in impingement syndrome.

A 12-year-old girl presents to the emergency room with worsening foot pain. 2 weeks ago, she stepped on a nail while wearing rubber-soled tennis shoes. The area was thoroughly cleaned, but she has developed worsening pain, redness, and drainage from the area. X-ray shows periosteal changes at the site of the wound. A wound culture is obtained. Question What organism is most likely to grow on culture for this specific mechanism of injury? 1 Staphylococcus aureus 2 Escherichia coli 3 Pseudomonas aeruginosa 4 Serratia marcescens 5 Salmonella enterica

3 Pseudomonas aeruginosa Pseudomonas aeruginosa is the most common cause of osteomyelitis when a penetrating object passes through the sole of the shoe, though Staphylococcus aureus is the most common cause of osteomyelitis overall.

A 21-year-old male patient presents with a 3-month history of pain in the left shoulder. He is a left-handed pitcher for the college baseball team. The pain began when he started spring training a few months ago, and it has gradually worsened. He is now having difficulty lifting the left arm above the head. He denies any injury to the shoulder, and he states that ibuprofen provides some relief. On physical exam, the patient has tenderness to palpation of the lateral left shoulder just under the acromion, limited abduction of the left shoulder, a negative drop-arm sign, and a negative crossover test. Question What condition best explains this patient's symptoms? 1 Bicipital tendinitis 2 Acromioclavicular arthritis 3 Rotator cuff tendinitis 4 Complete rotator cuff tear 5 Adhesive capsulitis

3 Rotator cuff tendinitis Rotator cuff tendinitis is most likely. Patients cannot lift the arm above shoulder level (abduction and external rotation) and have tenderness with palpation of the area under the acromion on the affected side. The rotator cuff muscles are the supraspinatus, infraspinatus, teres minor, and subscapularis (SITS muscles). The tendons of three muscles form a musculotendinous unit where they insert into the proximal humerus. Repetitive movements of the shoulder can lead to inflammation of the tendons of these muscles. Supraspinatus is most commonly affected. Drop-arm sign: the patient fully abducts the affected arm to shoulder level or up to 90°, and then lowers it slowly. If the patient cannot hold the arm fully abducted, the drop-arm test is positive. A positive drop-arm sign indicates a complete tear in the rotator cuff, not just tendinitis.

A 21-year-old man presents with a 3-month history of pain in his left shoulder. He is a left-handed pitcher for his college baseball team and states that the pain began at spring training and has gradually worsened since that time. In addition to the pain, the patient is now having difficulty lifting his left arm above his head. He denies any injury to the shoulder itself and states that ibuprofen provides some relief. You suspect rotator-cuff tendinitis. Question What physical exam finding would you expect to find in this patient? 1 A positive crossover test 2 Tenderness to palpation along the anterior aspect of the left humerus 3 Tenderness to palpation just under the acromion 4 A positive Speed's test 5 Palpable crepitus during range-of-motion maneuvers

3 Tenderness to palpation just under the acromion The muscles of the rotator cuff include the supraspinatus, infraspinatus, teres minor, and subscapularis, otherwise known as the SITS muscles. Three of these muscles—the supraspinatus, infraspinatus, and teres minor—insert on the greater tuberosity of the humerus. Repetitive movements of the shoulder, as with throwing or swimming, can lead to inflammation of the tendons of the rotator cuff muscles; the most commonly affected muscle is the supraspinatus, which inserts directly under the acromion. Classically, a patient with rotator cuff tendinitis will have tenderness to palpation just under the acromion on the affected side as well inability to lift the arm above shoulder level (abduction and external rotation).

A 25-year-old man presents with back pain and stiffness. He has noticed an increasing presence of stiffness and general fatigue along with the pain. The pain is much worse first thing in the morning, and he rates it a 7/10. Radiation occasionally occurs into the buttocks, and the symptoms improve with activity. Physical examination is remarkable for forward stooping of the thoracic and cervical spine and decreased lateral flexion in the lumbar spine. Question What is the most likely diagnosis? 1 Spondylolisthesis 2 Osteoarthritis 3 Rheumatoid arthritis 4 Ankylosing spondylitis 5 Psoriatic arthritis

4 Ankylosing spondylitis Ankylosing spondylitis (AS) is a chronic inflammatory disease associated with back pain and progressive spinal stiffness. Patients with AS are typically males ages 20-30 with transient or persistent peripheral arthritis and other manifestations like anterior uveitis. The pain is worse at night and in the morning and improves with physical activity. The goal of managing AS is to maximize long-term health-related quality of life: relief of symptoms, maintenance of function, prevention of complications from spinal disease, and minimization of extraspinal and extra-articular manifestations and comorbidities.

A 19-year-old man presents with pain and deformity of his right dominant shoulder after a sudden jerking movement to the same shoulder from a wrestling competitor approximately 1 hour ago. He states he felt a clunking sensation when it happened. He was unable to continue wrestling and has pain with movement of the right shoulder. Question What initial imaging should be the next step? 1 MRI of the shoulder 2 CT of the shoulder 3 Anterior/posterior and internally rotated humeral view radiographs 4 Anterior/posterior, scapular lateral, and axillary radiographs 5 Internal and external radiographs of the humeral head

4 Anterior/posterior, scapular lateral, and axillary radiographs The patient's history indicates a potential shoulder dislocation, with an anterior dislocation being more common. The initial imaging recommended to evaluate this diagnosis is radiographs with AP, scapular (Y view), and axillary views. An anterior shoulder dislocation can be seen on the AP view. A scapular (Y view) should be obtained to assess the position of the humeral head. There can be false positives on the scapular view, so axillary view should be performed. An axillary view taken with the arm in abduction is the best way to visualize a posterior dislocation.

An 18-year-old man presents 30 minutes after falling on his outstretched arm while skateboarding. He is guarding his left forearm near his wrist with his right hand, and he has his left arm against his body for support. There is a deformity similar to the image with edema and ecchymosis at the wrist. He is neurovascularly intact and the skin is closed. His left elbow and shoulder exam are normal. Question With this type of deformity, what fracture is most likely? 1 Smith fracture 2 Torus fracture 3 Salter-Harris IV fracture 4 Colles' fracture 5 Die-punch fracture

4 Colles' fracture

A 62-year-old man presents with vision problems and difficulty swallowing. Over the last week, he has had a constellation of symptoms; they began with numbness and tingling in his feet and progressed to weakness that now affects both lower and upper extremities. He has started to notice difficulty swallowing and double vision since yesterday. It feels difficult for him to take a big breath. His past medical history is non-contributory, and he takes no medications. Exam reveals bilateral absence of patellar and ulnar reflexes. A lumbar puncture is performed to confirm the diagnosis. Question What cerebrospinal fluid (CSF) finding is most likely? 1 Decreased CSF glucose content 2 Decreased CSF protein content 3 Elevated CSF polymorphonuclear cell count 4 Elevated CSF protein content 5 Elevated CSF lymphocyte count

4 Elevated CSF protein content Symmetrical ascending paralysis as described in this patient is indicative of Guillain-Barré syndrome (GBS). The cause of GBS is unknown, but it is generally thought to be an inflammatory autoimmune process. More than half of patients with GBS report an antecedent illness. The antibodies produced in response to antigens present in the infectious agent are thought to cross-react with components of human neurons, prompting an acute postinfectious demyelinating process. Lumbar puncture characteristically reveals elevated CSF protein content. Other results are normal, although the white blood cell count may be somewhat elevated.

A 32-year-old man presents with a severe headache; he has had 2 similar headaches within the past week. He describes a burning, "hot poker" type of pain located primarily behind his right eye. He notes that his eye waters profusely with the headache. His nose is initially congested, then it starts running. Only his right side is affected. The headache is so severe that he cannot work or sleep through it, and he is unable to concentrate on anything else. The headaches have been unresponsive to over-the-counter pain medications. The episodes seem to last about 1 hour. He denies any other symptoms. This patient has no chronic medical conditions, and he takes no regular medications. Question What is the most likely underlying pathophysiology of this patient's condition? 1 Antigen binds to IgE, leading to the release of histamines 2 Growth of a tumor near the base of the skull, leading to pressure on the cerebellum 3 Increased emotional stress, leading to contractions of head and neck 4 Increased trigeminal nerve and parasympathetic activity, leading to vasodilation 5 Reactivation of varicella-zoster virus, leading to inflammation in a ganglion

4 Increased trigeminal nerve and parasympathetic activity, leading to vasodilation

A 45-year-old African American man with no significant past medical history presents with a 1-hour history of left retro-orbital headache. The headache was of a sudden onset, and it began upon waking this morning. It is described as excruciating, stabbing, sharp, and lancinating; it is rated as severe in intensity. He denies any preceding infections, nausea, vomiting, fever, chills, focal weakness, numbness, tingling, hearing, gait, or speech changes. He recalls a similar episode several months ago; it lasted about 3 hours and dissipated without complications. His physical exam is remarkable for painful distress, nasal congestion with rhinorrhea, left ocular miosis, and left forehead flushing diaphoresis. Question What is an additional expected manifestation in this patient? 1 Cervical muscle tenderness 2 Temporal artery tenderness 3 Visual scintillations and scotomas 4 Lacrimation and conjunctival injection 5 Papilledema

4 Lacrimation and conjunctival injection

A 69-year-old woman suffered a massive stroke 6 weeks ago. She is now recovering from the stroke, but she has residual paralysis and sensory impairment of her right arm. She is also unable to speak and unable to turn her eyes to the right. Question The most likely site of her lesion is in the area that is supplied by what artery? 1 Right anterior cerebral artery 2 Left anterior cerebral artery 3 Right middle cerebral artery 4 Left middle cerebral artery 5 Right posterior cerebral artery

4 Left middle cerebral artery The left middle cerebral artery is responsible for motor function of the upper extremity and face, so upper extremity weakness and facial plegia are present with infarctions in this territory. Lower extremity weakness may be present, but it is less prominent. You can also see speech and vision impairment.

A 73-year-old man presents with the inability to actively raise his left non-dominant arm to reach his kitchen cabinet. This began a month ago after his shoulder pain improved. He had a history of pain in that shoulder for over 6 months that kept him from sleeping on the left side and the pain would often wake him. There was no specific injury he can recall, but he felt a pop a month ago while taking out the trash. Question What is the diagnostic study of choice if surgery is indicated? 1 CT without contrast 2 Electromyelogram 3 Ultrasound 4 MRI 5 Plain films

4 MRI This older man likely has a torn rotator cuff. If surgical treatment is being considered, MRI is the imaging study of choice because it can provide additional information on the status of the muscle and the size and location of the rotator cuff tear

A 70-year-old man with hypertension, diabetes, obesity, and coronary artery disease presents with a 3-day history of fever, headache, nausea, vomiting, lethargy, and myalgias. His wife states that the symptoms began the day after a summer evening walk around a lake. Today, his condition has worsened; he has developed a diminished level of consciousness, behavioral changes, and abnormal movements. His physical exam reveals a lethargic man with photophobia, cranial nerve deficiencies, neck stiffness, and abnormal extremity movements. Question Based on your suspected diagnosis, what is the next step in the management of this patient? 1 Non-contrast head CT scan 2 Monitoring of intraventricular intracranial pressure 3 Parenteral cefazolin 4 MRI of the brain 5 Lumbar puncture

4 MRI of the brain This patient's most likely diagnosis is West-Nile virus encephalitis, caused by a flavivirus mosquito-borne infection. Laboratory samples and blood cultures should be collected before the start of IV therapy. Even in uncomplicated cases of encephalitis, most authorities recommend a neuroimaging study in the form of a magnetic resonance imaging (MRI), the imaging of choice in suspected encephalitis cases. MRI may be completely normal, but it is the best way to rule out focal neurologic causes such as an abscess given this patient's neurologic symptoms.

A 27-year-old man presents with knee pain that started while he was playing basketball the previous afternoon. The patient states that he twisted his left knee with his left foot still planted on the floor as he attempted to pass the ball to a teammate. He describes a popping sensation at the time of the injury, but he could still bear weight on his leg. He does not recall any immediate swelling. The morning of presentation, the patient noted swelling of the knee and pain that was significantly worse than it was the day before. He has taken 600 mg ibuprofen for the pain but has experienced only mild relief. Upon examination of the left knee, there is a limitation in range of motion, the presence of medial joint line tenderness, and an effusion. McMurray's test is positive. Question What is the optimal imaging modality or procedure for confirming the suspected diagnosis? 1 Anteroposterior (AP) and lateral X-ray 2 Ultrasonography 3 Computed tomography (CT) scan 4 Magnetic resonance imaging (MRI) 5 Positron-emission tomography (PET) scan

4 Magnetic resonance imaging (MRI) This patient most likely has a medial meniscal injury of the left knee. Meniscal tears are the most common knee injuries encountered in primary care. They result from a twisting action exerted on the knee joint while the foot is still in a weight-bearing position. Magnetic resonance imaging (MRI) is still the optimal test for confirming the diagnosis of a meniscal injury. All intra-articular structures (menisci, ligaments, and articulating surfaces) can be visualized in great detail. Patients with meniscal injuries often report a tearing or popping sensation at the time of injury, followed by severe pain. Effusions associated with these injuries accumulate over hours; this is in contrast to ligamentous injuries in which hemorrhage causes immediate swelling. The knee swelling and pain associated with meniscal tears are typically worse the day after the injury. With medial meniscus injuries, tenderness localized to the medial joint line and reduction in range of motion may be noted on exam. McMurray's test can be performed to aid in the diagnosis.

A 20-year-old male college student presents with fever, chills, malaise, headache, photophobia, confusion, and numerous petechiae on his extremities and trunk. On examination, he has positive Brudzinski and Kernig's signs. A CT scan of the head is within reference limits. A lumbar puncture reveals increased leukocytes, particularly polymorphonuclear neutrophils, increased protein, and decreased glucose levels. Gram staining of the CSF reveals Gram-negative cocci in pairs. Question What is the most likely causative agent for the infection in this patient? 1 Bordetella pertussis 2 Francisella tularensis 3 Haemophilus influenzae 4 Neisseria meningitidis 5 Yersinia pestis

4 Neisseria meningitidis

A 33-year-old Caucasian man with a history of severe type 1 diabetes presents due to a red swollen pinky toe on his left foot. He was running to answer his home phone and stepped on an exposed carpet tack 2 days ago. He washed it out and covered it as soon as he could. Since then, he has noted a significant increase in redness and severe restriction of movement of this toe; these characteristics have worsened tremendously in the last 12 hours. In addition, he admits to "feeling feverish" for the past day, and he experienced significant night sweats the previous night. He denies formally measuring his temperature. He has tried to take acetaminophen 500 mg, 2 tablets every 8 hours without any noticeable relief. Pertinent physical examination findings include an oral temperature of 102.8°F, significant erythema and edema extending the entire fifth toe of the left foot accompanied by restricted range of motion due to tenderness and swelling. There is a small amount of purulent drainage at the site of the wound. Question What is the most likely diagnosis? 1 Osteoarthritis 2 Gouty arthritis 3 Osteopenia 4 Osteomyelitis 5 Psoriatic arthritis

4 Osteomyelitis

A 34-year-old man presents with a 2-day history of right ankle pain and swelling. He reports experiencing discomfort with bearing weight, ambulation, and when driving a car. On further questioning, he denies experiencing a recent trauma, although he does recall spraining his ankle approximately 1 year ago. The patient is not married and is heterosexual; he usually uses condoms as contraception, "but not every time." On physical examination, the patient's temperature is 99.9°F. His right ankle shows swelling, is warm to palpation, and reveals an effusion. With passive range of motion of the right ankle, significant pain is elicited. Question What is the most appropriate next step in the management of this patient? 1 Check serum uric acid level; if elevated, initiate therapy with indomethacin and colchicine. 2 Obtain plain radiograph of the right ankle to assess for chondrocalcinosis. 3 Obtain plain radiograph of the right ankle to assess for structural damage. 4 Perform arthrocentesis of right ankle with analysis of synovial fluid. 5 Treat with ibuprofen and have patient return to office in 1 week for follow-up.

4 Perform arthrocentesis of right ankle with analysis of synovial fluid. The acute onset of symptoms, low-grade fever, and lack of trauma in this patient warrant a prompt evaluation. Empirical therapy will not provide a definitive diagnosis and could potentially result in a serious illness such as septic arthritis being missed. The most appropriate initial evaluation of a patient with monoarticular arthritis is arthrocentesis. Joint aspiration should be performed with aseptic technique as a part of the evaluation of every case of acute monoarthritis. Analysis of the synovial fluid includes a WBC count, differential, appropriate cultures and stains for microorganisms, and polarized-light microscopy. The WBC count in the synovial fluid is useful in distinguishing inflammatory from non-inflammatory arthritis; levels greater than 2000/mm3 are consistent with inflammation. Gonococcus (Neisseria gonorrhea) is the most common cause of infective arthritis in young adults. In this particular case, the ankle is swollen due to gonococcal arthritis. Gonorrhea is the most frequently reported communicable disease in the US. Disseminated gonococcal infection is most often the cause of acute septic arthritis in sexually active adults and the reason for most hospitalizations due to infective arthritis.

A 75-year-old woman with a past medical history of hypertension, hyperlipidemia, and obesity presents with insidious but progressive bilateral knee pain for the past several months. She states that her pain is worsened with movement and relieved by rest. She reports "cracking" sounds in her knees upon ambulation and minimal stiffness lasting 10 minutes. Her physical exam is remarkable for bilateral knee crepitus, joint line tenderness, and an antalgic gait; there is no swelling, erythema, or warmth noted. Question What is an expected diagnostic test result in this patient? 1 Elevated erythrocyte sedimentation rate 2 Juxta-articular demineralization on x-rays 3 Negatively birefringent crystals 4 Plain radiograph subchondral sclerosis 5 Synovial fluid leukocytosis

4 Plain radiograph subchondral sclerosis This patient is presenting with signs and symptoms consistent with osteoarthritis. Knee radiography is typically expected to reveal joint space narrowing (especially of the medial compartment), thickened, dense subchondral sclerosis, osteophytes, and bony spurring. Other findings include subchondral cyst formation and lipping of marginal bone.

A 24-year-old man with no significant PMH presents due to increasing pain in his right groin and buttock and difficulty walking. The pain has been present and worsening for about 1 month. He further reports that he had significantly injured himself (also near his right hip) in a fall while snowboarding about 3 months ago. He did not seek immediate care and had difficulty ambulating for a week, then saw a chiropractor to "put his lower back back in." The pain and gait problems eventually improved. Now he is concerned because the pain has returned with no new history of trauma. He denies any family history for musculoskeletal and rheumatologic conditions. On physical exam, the patient was noted to walk with a slight limp. Examination of the hip, buttock, and groin region reveals no edema, erythema, or ecchymosis. The exam does not produce one particular point of tenderness with palpation, passive range of motion is limited and painful, especially with forced internal rotation. A straight-leg raise against resistance elicits pain. Question What diagnostic study would confirm this patient's condition? 1 Arthroscopy 2 Bone biopsy 3 Nuclear bone scan 4 Plain radiography (X-ray) 5 Ultrasound

4 Plain radiography (X-ray) This patient's condition is suspicious for avascular necrosis (or the more preferred term, osteonecrosis) of the hip. Avascular necrosis (AVN) is a relatively common complication, following a traumatic hip dislocation, when the vascular supply to the femoral head is damaged and bone death occurs. There are many theories about the exact mechanism; other causes such as vascular, congenital, and autoimmune diseases have been implicated with osteonecrosis. Plain radiography (X-ray) is the preferred initial test, and if the "crescent sign" is seen, AVN is confirmed. The crescent sign is a lucent (dark-appearing) crescent-shaped region of the femoral head, usually just a sliver beneath a more opaque (white-appearing) bone edge, which indicates some sclerosis and bone structure collapse upon itself.

A 73-year-old man has had shoulder pain for more than 6 months and it has kept him from sleeping on his left side and has often woken him up. He presents with an inability to actively raise his left arm that started 1 month ago, before which his shoulder pain had improved. There was no specific injury that he can recall, but he felt a pop in the shoulder a month ago while taking out the trash. Question What finding do you expect on physical exam? 1 Ecchymosis over the deltoid 2 Normal strength of all shoulder muscles 3 Negative (-) painful arc sign 4 Positive (+) drop arm test 5 Paresthesia of the median, radial, and ulnar nerves

4 Positive (+) drop arm test This presentation indicates a rotator cuff tear; drop arm test is positive in this condition. Drop arm test is positive when the patient is unable to control shoulder adduction. History typically includes pain for several months, with difficulty sleeping on the affected side and waking from the pain. Active range of motion is limited, but passive range of motion is usually full (unless associated with a frozen shoulder prior).

A 28-year-old female patient presents with severe right ankle pain after a fall on ice. The patient cannot move the ankle. She states that it is exquisitely tender and the pain worsens with movement or when attempting to bear weight. She states that she was immediately unable to take any steps due to how substantially painful the right leg is. During the physical examination, the patient still cannot bear weight on the injured leg. The right ankle is painful to even light palpation throughout the malleolar zone. The skin is a healthy color, and there are no signs of lacerations or tenting. Pulses are palpable. The entire area of the right ankle is swollen and appears dislocated. The left leg, foot, and ankle are unremarkable. Question What is the most likely diagnosis? 1 Right inversion ankle sprain 2 Right eversion ankle sprain 3 Right ankle strain 4 Right ankle fracture 5 Right ankle hematoma

4 Right ankle fracture This patient most likely has a right ankle fracture. Since there is significant swelling bilaterally, the tibia and the fibula are both highly likely to be fractured. Key features of an ankle fracture include pain, swelling and signs of bruising after the traumatic event, apparent deformity, and inability to bear weight. Ottowa Ankle Rules have been found to determine with high sensitivity if an ankle fracture can be ruled out. These rules state that an ankle x-ray is only required if there is any pain in the malleolar zone and if at least one condition is met:

A 3-year-old black female patient with sickle cell disease presents with a limp; her mother first noticed it yesterday morning. The girl has not had fever, nausea, or vomiting, but she is in pain. Her sickle cell disease has been relatively mild; she is fully immunized and takes prophylactic oral penicillin 125 mg 2 times a day. On exam, she is afebrile. There is pain in the left hip on internal and external rotation, but there is no definite point tenderness. Exam of the left knee and right hip are negative. The provider is concerned about septic arthritis and plans further workup. Question What organism is the provider most concerned about? 1 Staphylococcus aureus 2 Streptococcus species 3 Haemophilus influenzae type b 4 Salmonella species 5 Neisseria gonorrhoeae

4 Salmonella species

A 22-year-old woman presents for evaluation of knee pain. The patient appears in moderate distress and is unable to fully bear weight as she moves to the exam table. Her physical exam reveals a swollen and tender left knee. She is tender at the lateral femoral condyle, lateral tibial plateau, and tibiofemoral joint line. Lachman and anterior drawer tests are positive. Posterior drawer, Apley grind, patellar grind, and McMurray tests are negative. There are no sag or apprehension signs. Valgus and varus stress tests are normal. An X-ray shows no fractures. Question What inciting event is most likely in this patient's history? 1 Dashboard knee injury in a car collision 2 Fever and prior Neisseria gonorrhoeae infection 3 Frequent squatting and climbing stairs 4 Sudden landing playing basketball 5 Lateral blow to knee

4 Sudden landing playing basketball This patient's physical exam findings demonstrate a deficiency in the anterior knee, specifically the anterior cruciate ligament (ACL). ACL injuries most often occur acutely while playing sports, but they can occur with other mechanisms. Female sex is a risk factor. The patient may give a history of hearing a "pop," acute pain, and buckling or instability of the knee. Sudden landing after jumping during basketball is the most likely to produce an ACL injury. A dashboard injury to the knee during a motor vehicle collision could produce several injuries, but it would be most likely to produce a posterior dislocation and disrupt the posterior cruciate ligament (PCL). A PCL injury would likely yield physical exam findings of a positive posterior drawer and sag sign (the anterior tibia sags posteriorly). Fever and prior Neisseria gonorrhoeae infection with knee pain suggests septic arthritis as the cause. Septic arthritis would produce swelling and tenderness, but it would not be expected to demonstrate instability as seen in positive anterior drawer and Lachman tests. Frequent squatting and climbing stairs can contribute to knee pain and would be suggestive of a patellofemoral syndrome or possibly osteoarthritis (the latter is unlikely in a 22-year-old patient). If this patient's knee pain had been caused by squatting and stair-climbing, her physical exam might have demonstrated a positive apprehension test and patellar grind test. A lateral blow to the knee is associated with disruption of the medial and/or lateral collateral ligaments. The physical exam to have a positive valgus and/or varus stress test. A lateral knee injury would not typically cause ACL injury.

A 45-year-old female patient presents with a 2-week history of almost daily headaches. She describes feeling as if a rubber band is around her temples, making it extremely hard to concentrate on completing tasks. She denies auras, nausea, vomiting, or specific neurological issues, but she does describe some sensitivity to excessive noise and glaring lights. She has not been able to identify any obvious triggers. Question What is the most likely diagnosis? 1 Cluster headache 2 Migraine headache 3 Post-traumatic headache 4 Tension headache 5 Giant cell arteritis headache

4 Tension headache

A 36-year-old G1P1001 woman presents with a 4- to 5-week history of pain along her right lateral wrist; the pain worsens when she tries to grasp something. There was no trauma. She gave birth 6 weeks ago and carrying her baby is difficult secondary to pain. She is right-handed and has never had any issues like this before. On exam, there is tenderness and edema over the radial styloid. You then have the patient fully flex her thumb, adduct, and grasp it with that hand. You then place her hand in ulnar deviation, which reproduces the pain described above. Question What is the most likely diagnosis? 1 Rheumatoid arthritis 2 Ganglion cyst 3 Scaphoid fracture 4 de Quervain tenosynovitis 5 Radial nerve entrapment

4 de Quervain tenosynovitis de Quervain Tenosynovitis is a tenosynovitis of the abductor pollicis longus and extensor pollicis brevis tendons at the radial styloid process. It is most commonly seen in women 35-55. It can happen when women carry children with their thumb overextended. On exam, there can be tenderness and swelling over the radial styloid. There is also a positive Finkelstein test, which is what is described in the scenario.

A 44-year-old man starts to notice that his eyelids are drooping. Some time afterward, his jaw becomes weak. He has difficulty swallowing and experiences weakness in his limbs. He is quite embarrassed when he eats because he must use his hand to help support his jaw. His weakness gets progressively worse. Finally, he seeks medical attention. His physical examination demonstrates the weakness in his limbs, but no sensory defects are present. A Tensilon test is done and is positive. Question What is the most likely underlying pathology of these symptoms? 1 Inhibition of acetylcholine release 2 Blockage of the sodium channels 3 Demyelination 4 Subacute combined degeneration of the spinal cord 5 Antibodies to the acetylcholine receptor

5 Antibodies to the acetylcholine receptor This patient has myasthenia gravis. Ocular muscle weakness, ptosis, dysphagia, and limb weakness can all be seen with myasthenia gravis. When the initial symptom is ocular weakness, Eaton Lambert syndrome is extremely unlikely. Eaton Lambert syndrome tends to not involve the extra-ocular muscles or the muscles involving chewing, swallowing, or speech.

A 45-year-old man goes to a party and enjoys several glasses of an alcoholic cocktail. His past medical history is significant for headaches. The drinks trigger a unilateral right-sided headache. The headache is behind his right eye and spreads to his forehead. He also notices that his right nostril has a watery discharge and his right eye is tearing. He describes the pain as if he were "being stabbed in my eye." Question What is the most likely diagnosis for this patient? 1 Tension headache 2 Hangover headache 3 Common migraine 4 Classic migraine 5 Cluster headache

5 Cluster headache

A 16-year-old boy presents with acute onset of stiff neck, fever, headache, and vomiting. On exam, he appears lethargic, has limited range of motion of his neck, and a petechial rash is noted. Fever is 103°F. The patient's medical history is non-contributory. Question What is the most likely etiology of the suspected diagnosis? 1 Enterovirus 2 Varicella virus 3 Syphilis 4 Pneumococcus 5 Meningococcus

5 Meningococcus Meningococcal meningitis is a life-threatening infection caused by the Neisseria meningitidis bacterium. Most cases occur in children and young adults. The onset is often abrupt, and early treatment is essential to achieving reduced morbidity and mortality. In addition to the usual symptoms, a petechial rash (non-blanching red-brown spots on all parts of the body) is characteristic and caused by capillary leak and platelet dysfunction. Routine vaccination against meningococcus is recommended for adolescents in the United States; the first dose at age 11 or 12, and a booster dose at age 16. If this patient has not had routine medical care for many years, then he will not have been vaccinated against this disease.

A 32-year-old man presents with pain in his back, buttocks, and posterior thighs for 2 days after lifting a heavy load at work. He denies pain when sleeping unless he rolls over in bed. He also has pain with just standing or sitting still in a chair. He denies any radicular symptoms or bladder or bowel dysfunction. Question What diagnostic test does this patient require? 1 MRI of the LS spine 2 CT of the LS spine 3 Plain radiographs of the LS spine 4 Electromyogram 5 No diagnostic tests required

5 No diagnostic tests required Most patients presenting with low back pain less than 4 weeks without evidence of neurologic compression do not require any diagnostic tests. Current recommendations by the American College of Physicians advise against routine imaging in patients presenting with low back pain unless neurologic deficits are present. This patient denies any nocturnal symptoms, radicular symptoms, bladder or bowel dysfunction, so imaging is not indicated.

A 63-year-old man presents with pain in both knee joints that began almost 1 year ago. It was mild and present only during activity, but it has worsened and is present during rest. There is no joint stiffness. He has diabetes and has been on insulin treatment for 10 years. BMI is 30. Vitals are within reference ranges. Examination reveals pale conjunctivae. On examination of the knee joints, crepitus is present and range of motion is reduced, but there is no tenderness, warmth, or erythema. Osteoarthritis is suspected and lab investigations and x-ray are ordered. Meanwhile, exercises and ibuprofen as a pain reliever are prescribed. Question What is a risk factor for osteoarthritis in this patient? 1 Sex 2 Anemia 3 Diabetes 4 Insulin 5 Obesity

5 Obesity

A 72-year-old woman with a past medical history of COPD, deep venous thrombosis, breast cancer in remission, hyperthyroidism, and premature ovarian failure presents with sudden-onset severe lower back pain of 2 hours' duration. She states that the pain began when stepping onto the floor as she got out of bed. She denies any falls, prior injuries, genitourinary problems, or lower extremity numbness, tingling, paresthesias, or weakness. She takes a daily prednisone tablet for COPD. Her social history is notable for a sedentary lifestyle and a 40 pack-year smoking history. Her physical exam reveals a tall thin-framed woman with noticeable lid lag. Her thoracic vertebral body is exquisitely tender at T12, and she has limited mobility of all thoracolumbar planes of motion. Lateral and AP lumbar spine X-ray reveals osteopenia and collapse of vertebral body of T12. Question What is the best next step in managing this patient? 1 Increase the dose of prednisone to reduce symptomatology. 2 Order human osteocalcin and bone alkaline phosphatase levels. 3 Perform a quantitative computerized tomography of the spine. 4 Prescribe estrogen replacement therapy. 5 Order a dual-energy X-ray absorptiometry test.

5 Order a dual-energy X-ray absorptiometry test. This patient's symptoms and the radiographic images suggest osteoporotic vertebral compression fracture of the T12 vertebral body. Risk factors for primary osteoporosis include early menopause or premenopausal estrogen deficiency states, prolonged periods of inadequate calcium intake, sedentary lifestyle, and tobacco and alcohol abuse. The most widely used techniques for assessing bone mineral density (BMD) are dual-energy X-ray absorptiometry (DEXA) and quantitative computerized tomography (CT).

A 35-year-old man with no significant past medical history has been experiencing progressive, moderately severe right knee pain for the past 3 months. Pain was initially felt only at night, but it is now constant throughout the course of the day for the last several weeks. It is especially severe upon ambulation and during knee ranges of motion, causing him to limp. He denies fever, chills, weight changes, history of gout, sexually transmitted diseases, hip or back pain, recent instrumentation, trauma, or injuries. His physical exam reveals an antalgic gait with limp, limited ranges of motion of the right knee, and a 3 cm diameter firm, tender mass at the distal femur. There is no erythema, crepitus, alignment deformity, or effusion noted. Question What is the next appropriate step in the management of this patient? 1 Order a serum uric acid level and prescribe indomethacin. 2 Wrap the right knee in an elastic bandage and prescribe physical therapy. 3 Perform a joint aspiration with microscopic cellular assessment. 4 Obtain a blood culture and begin empiric dicloxacillin. 5 Order an LDH, ALP, and a plain radiograph of the knee.

5 Order an LDH, ALP, and a plain radiograph of the knee. This patient's manifestations are most suggestive of osteosarcoma. It is typically seen in patients in their teens and 20s, occurs more frequently in men than in women, and is found in the metaphyseal areas of long bones, with 50% of lesions seen about the knee joint. The distal femur is the most common site, followed by the proximal tibia, and then the proximal humerus. The only blood tests with prognostic significance are lactic dehydrogenase (LDH) and alkaline phosphatase (ALP). Patients with an elevated ALP at diagnosis are more likely to have pulmonary metastases. In patients without metastases, those with an elevated LDH are less likely to do well than are those with a normal LDH. Other laboratory tests include a complete blood cell (CBC), including a differential and platelet level, electrolytes, and liver and renal function tests. The practitioner should also obtain plain films of the suspected lesions in two views. Elevation of the periosteum may appear as the characteristic Codman triangle; in approximately 60% of cases, extension of the tumor through the periosteum may result in a "sunburst" appearance.

A 66-year-old woman presents with back pain. She has a 5-month history of taking prednisone and has been a smoker for the last 15 years, which she has difficulty quitting. Upon physical exam, her back is very tender upon palpation. When the patient walks, she appears very bent over and unable to stand upright. A laboratory finding on her DXA was a T score of -3.1. X-ray was conducted as well, and demonstrates a small fracture in her back. Question What is the most likely diagnosis? 1 Osteomalacia 2 Osteopenia 3 Osteoarthritis 4 Osteosarcoma 5 Osteoporosis

5 Osteoporosis

An 8-year-old boy sustained a puncture wound to his right foot by stepping on a nail through his sneaker 4 days ago. His mother said the wound bled profusely but the nail did not go completely through his foot. They washed the wound at home with soap and water, wrapped it in a bandage, and did not seek further care. This morning, he complained that it was very painful and his mother noted that his foot looked red and swollen. On exam, his temperature is 99°F; pulse is 114 BPM, and his BP is 104/68 mm Hg. The plantar surface of his right foot has a small 2 mm scabbed entry wound that is surrounded by a 5-6 cm area that is erythematous, swollen, and quite tender. There is a scant amount of thin seropurulent material from the entry wound on examination. Question What bacteria is most likely the cause of the suspected complication the patient is experiencing? 1 Streptococci 2 Eikenella 3 Pasteurella 4 Clostridium 5 Pseudomonas

5 Pseudomonas Pseudomonas aeruginosa is the most common cause of foot osteomyelitis in plantar puncture wounds while wearing tennis shoes. It should be considered a likely pathogen when there is a penetrating injury through footwear, especially sneakers or from soil and manure contamination. It is believed that the penetrating object may push particles of foam contaminated with bacteria into the wound. In addition, anaerobes, Klebsiella, Bacteroides, Serratia, and Salmonella sp. are less common organisms in this setting that may also cause secondary infections.

A 35-year-old man presents with right shoulder pain that is becoming progressively worse. Although he visits the gym 3 times a week, over the past month he has not been able to increase the amount of weight he lifts secondary to the shoulder pain. He has not tried anything to alleviate the pain. He reports that the pain is at its worst at night while he is trying to sleep. He also reports pain while in the shower washing his hair, or when using the shoulder press machine at the gym. He denies any history of recent trauma or sports-related injury. Upon questioning, however, he reports that about 1 month ago he and his wife painted their entire house in one weekend. On exam, there is no notable swelling, atrophy, redness, or bruising of the shoulders. Point tenderness is noted over the right lateral deltoid muscle. Active ROM of the right shoulder at 80° of abduction elicits pain. Patient has a negative drop arm test, negative apprehensive test, and a positive Neer impingement test of the right shoulder. Question What is the most likely diagnosis? 1 Acromioclavicular sprain 2 Adhesive capsulitis 3 Fracture of the clavicle 4 Glenohumeral dislocation 5 Rotator cuff tendonitis

5 Rotator cuff tendonitis The most likely diagnosis, rotator cuff tendonitis (tendinopathy), is associated with pain and difficulty abducting or rotating the arm. Degree of trauma depends on the type of injury. With full thickness tears, the degree of trauma is significant (e.g., a major fall, MVA, shoulder dislocation). Partial thickness tears can be caused by lesser degree of trauma; overuse injuries can cause tears, as well. Tendonitis injuries are often associated with overuse. Symptoms are usually pain and weakness made worse by overhead activities, overhead abduction, and external rotation. Pain is usually located over the anterior and lateral aspects of the shoulder and may radiate into the lateral deltoid. The pain may worsen with sleeping on the affected extremity. The patient may describe a "catch" as the arm is brought into the overhead position. In addition, a typical painful arc usually occurs between 70 and 110° of abduction along with a positive impingement test, which is forced forward elevation of the arm. The pain results as the greater tuberosity impinges on the acromion. The drop test is associated with rotator cuff tears.

A 55-year-old woman presents with weakness, bone pain, and lethargy. A 24-hour urine sample reveals a spike of M protein. Based on the most likely diagnosis, what additional finding on bone marrow aspirate would support the diagnosis? 1 Teardrop-shaped erythrocytes 2 Lewy bodies 3 Schistocytes 4 "Starry sky" pattern 5 Russell bodies

5 Russell bodies Russell bodies are the accumulation of immunoglobulin in plasma cells: a characteristic finding in patients with multiple myeloma.

A 32-year-old man presents with pain in his right shoulder. He tells you he plays softball every weekend and does a lot of shoveling around his house. The shoulder has been sore for some time, but now it hurts to the point where he tries to avoid using it. The drop arm test is negative, and there is no redness, warmth, or obvious swelling, but the patient is unable to lift his arm up to 90° without pain. Question What is the most likely diagnosis? 1 Bursitis 2 Fibromyalgia 3 Gout 4 Rotator cuff tear 5 Tendinitis

5 Tendinitis Tendinitis is an inflammation of the tendon tissue, or the tendon sheath (tenosynovitis). The cause is often unknown. As the vascularity of tendons decreases with age, the incident of tendinitis will increase. Symptoms usually include painful tendons on movement. Swelling can be seen or felt on palpation. Along the tendon, localized tenderness of variable severity is present on palpation. Generally, there is not the warmth or redness of bursitis. During the drop arm test, the patient is asked to fully abduct the arm (straight out laterally from the side), then slowly lower the arm to the side. If they are unable to hold the arm against gravity, the drop arm test is positive. The drop arm test detects whether or not there are any tears in the rotator cuff.

A 20-year-old female college student presents to the student health center with a 1-week history of daily headaches. She has no significant past medical history. Upon further questioning, she admits to headaches occasionally over the past 2 years, but it is lasting longer this time. She classifies the pain as a 4/10 that is generalized, but is worse in the back of the head. The headaches are not debilitating or throbbing, and she describes it as an annoying pain. Acetaminophen helps somewhat but does not make the headache go away completely. On physical exam, there are no neurologic deficits, vision is 20/20 uncorrected, and vital signs are within normal limits. She does note tenderness upon palpation of the muscles of the head, neck, and shoulders. Question What is the most likely diagnosis? 1 Cluster headache 2 Giant cell arteritis 3 Intracranial mass 4 Migraine headache 5 Tension headache

5 Tension headache

A 23-year-old woman presents due to a headache that appears to be bilateral, and she describes it as a "tight" feeling around her head. She started physician assistant school 2 months ago and has been really stressed out. Upon physical exam, no abnormal findings are found. Additionally, all laboratory findings are within normal limits. Question What is the most likely diagnosis? 1 Cluster headache 2 Migraine with aura 3 Migraine without aura 4 Post-traumatic headache 5 Tension headache

5 Tension headache

A 37-year-old Hispanic man presents with a 4-month history of mild-to-moderate headaches; on average, he gets them 3-4 days per week. He has tried over-the-counter analgesics with minimal relief. He is seeking care now because he had been promoted to store manager several months prior to presentation, and he is worried that his headaches are affecting his concentration. His headaches are generalized in location, described as starting at the base of his head and extending all over, feeling "tight" in nature. He denies memory loss, photophobia, nausea/vomiting, rhinorrhea, lacrimation, and upper respiratory symptoms associated with the headaches. He also denies seizures, syncope, incoordination, vertigo, weakness, and paresthesias. The patient mentions his concerns for his work several times. Although he enjoys his work, he admits to having some anxiety about being able to handle his new duties. His family history is negative for headache. Physical exam is performed; vitals, neurological, cardiovascular, and HEENT findings are all normal. Question What is the most likely diagnosis? 1 Cluster headache 2 Headache secondary to brain tumor 3 Migraine headache 4 Subarachnoid hemorrhage 5 Tension headache

5 Tension headache

A 50-year-old obese woman presents with severe left knee pain. She states the pain began about 8 months ago but has grown significantly worse in the last 3 months. The patient denies any trauma or event that initiated the pain. She notes stiffness in the knee first thing in the morning that lasts around 5-10 minutes. The knee pain is worsened with activity and is relieved with rest. The patient's medication list includes lisinopril 10 mg once daily for high blood pressure. She has a documented medication allergy to acetaminophen; she states this makes her break out in hives. Physical examination findings reveal a Caucasian female with a BMI of 40. There is limited range of motion of the left knee and severe crepitus. Question Considering the most likely diagnosis for this patient, what lifestyle modification would be most beneficial in relieving her symptoms? 1 Bed rest 2 Blood pressure control 3 High impact activity 4 Screening labs 5 Weight loss

5 Weight loss

A 49-year-old right-hand dominant woman presents with a 2-week history of progressive pain in her right thumb and wrist area. She says that her thumb seems to "stick" in place upon movement. She states that she has never experienced this before. You ask her if she has been using her hands more often than normal, and she tells you she recently began to crochet a sweater for a family member. On physical examination, her pain is markedly exacerbated when she places her thumb into the palm of her hand, and when you passively move her hand in an ulnar direction. Question What is the most likely diagnosis? 1 Wrist arthritis 2 Scaphoid fracture 3 Dorsal wrist ganglion 4 Thumb arthritis 5 de Quervain's tenosynovitis

5 de Quervain's tenosynovitis The clinical picture is suggestive of de Quervain's tenosynovitis. It occurs commonly in middle-aged women, precipitated by repetitive use of the thumb. The inflammation thickens the tendon sheath abductor pollicis longus and extensor brevis on the thumb side of the wrist. This can cause pain, swelling, and may cause the thumb to lock in place upon movement. The Finkelstein test (full flexion of thumb into palm followed by ulnar deviation of the wrist) will produce pain and is diagnostic of de Quervain's tenosynovitis.


Kaugnay na mga set ng pag-aaral

Air Equipment Preparation Introductory Course (EPIC)

View Set

NCLEX Pediatrics Neuro and Musculoskeletal

View Set

Laboratory Management Midterm Quizzes

View Set

Chapter 21: OSPF Network Types and Neighbors

View Set